Sie sind auf Seite 1von 23

Rigorous Analysis…

SP Neelam, B Tech (IT BHU), M Tech (IIT Roorkee)

1. A homogeneous rod AB of length l = 1.8 m and mass t=t Fv


m is pivoted at the centre O in such a way that it can
rotate freely in the vertical plane, see the figure. The
N
rod is initially in the horizontal position. An insect S of Fh  f
N
the same mass m falls vertically with speed v on the x f

mg
point C, mid way between the points O and B. mg
Immediately after falling, the insect moves towards the 
end B such that the rod rotates with a constant angular
velocity . For the torque on the rod about the pivot O to be zero,
(a) Determine the angular velocity  in terms of v and l. the normal force N  on the rod by the insect must be
(b) If the insect reaches the end B when the rod has equal to zero. And in that case, generally speaking, the
turned through an angle of 90 o, determine v.
frictional force f will also be equal to zero.
(Take g = 10 m/s2.) (IIT-JEE 1992, 8 Marks)
Now look at the forces acting on the insect. The normal
S (Insect) force N   0, the frictional force f  0. So the only
force of the insect is its weight mg. The insect starts
v moving towards the end B of the rod immediately after
falling on it. Under these conditions the path followed
l l
by the insect must be somewhat as shown in the figure.
A O 4 4 B A

Pivot C

Solution : The mass of the insect is equal to the mass of


a 1.8-m long rod. A very large insect indeed! Large P

insects have masses 50-70 grams. But their lengths vary Path of the Insect
from 10 to 15 centimeters. Here, the insect has to be
treated as a particle. The clauses ‘…falls vertically with B
speed v on the point C’ and ‘…reaches the end B’ will
assume meaning only in that case. If this is so, how can the insect reach end B when the
Also, the motion of the ‘rod-insect’ system, or only rod rod has turned through 90 angle? So if the rod were to
or only insect for that matter, must be governed by the rotate with a constant angular velocity the insect would
laws of Particle Dynamics. The forces on the system, or be moving as a projectile under gravity, taking the
on its parts, are the usual forces we study - the weight, parabolic path as shown in the figure!
the normal force, the frictional force, etc. However, a
But there can be a frictional force on the insect, and its
rigorous analysis shows that unknown forces are
present here. The insect makes special efforts. The reaction on the rod, without the normal force N 
motion of the insect is in no way related to the motion shown in the figure above. The insect may press the rod
of the rod except that the insect somehow remains in on its sides, and thereby generate the required frictional
contact with the rod and moves towards its end B. force. This is certainly possible. But then the insect
must somehow balance the component of its weight
First let us apply the laws of Particle Dynamics. normal to the rod. There must not be any normal force
After falling on the rod the insect moves towards the on the rod by the insect.
end B, such that the rod rotates with a constant angular Also, for the insect, the force equation in the radial
velocity. This implies that the angular acceleration of
direction is f  mg sin   m(2 x  arel ).
the rod about the pivot is zero. And this will happen if
the torque on the rod about the pivot is zero. Here, arel is the acceleration of the insect relative to the
Consider the forces on the rod when it rotates about its rod. From this equation it can be seen that the force f
pivot. varies in a very complicated way during the motion of
the system.

1♦ Science Pyramid ♦
Hence, the insect makes special efforts in generating d  ml 2  
unknown forces that cannot be accounted for. mgx cos     mx 2  
dt  12  
Let us ignore all these details, treat the insect as a
particle, ignore the torque of the unknown force about dx
the pivot that the insect somehow generates if any, and or mgx cos   2m x 
dt
solve the problem as given below.
The forces acting on the ‘rod-insect’ system during the dx
or g cos   2 . …(1)
impact is shown in the figure. The normal force on the dt
rod by the pivot during the impact is an impulsive This is the differential equation of motion of the
force, N >> 2 mg. What all information do you need to system. Substituting  = t in this equation,
calculate the value of the normal force N during the
dx
impact? g cos t  2
dt
N
or g cos t dt = 2dx. …(2)
A O B t=t N

mg mg  = t
 
Here Fext  0 , (convince yourself), so linear
x 
momentum of the ‘rod-insect’ system changes. mg
  
From equation   dL / dt , you get L = constant if mg
 
 = 0. The normal force N on the rod by the pivot is a
force not known to you. Can you locate a point about The rod turns through /2 in time (/2)/ and the
which the torque acting on the system is zero unless position of the insect changes from x = l/4 to x = l/2.
you know all the forces acting on the system?
l  l Integrating both the sides of equation (2) with these
Also, clearly,  pivot  mg.  0. Here  mg   t  0 limits
4  4
 / 2 l/2
as t is negligibly small. So under ‘impulse 0 g cos t dt   2 dx
l /4
approximation’ the angular momentum of the 'rod-
insect' system about the pivot can be conserved. (Why g
or | sin t |0 / 2  2 | x |ll /2
/4
?) 
l  ml 2 l 
2 g    
or or sin .  sin 0   2   
mv 
4  12
 m  
 4     2  2 4
g l
l  1 1 2 or  2
or mv     ml   4
4  12 16 
2g
12 v or 2 
or  . l
7 l 2
The insect moves towards end B such that the rod  12 v  2g  12 v 
or      
rotates with constant angular velocity. There will  7 l l  7 l
always be a torque on the system about the pivot. This 7 7
torque does not result in an angular acceleration or v  2 gl  2 10  1.8  3.5 m/s.
  12 12
otherwise  will change. The equation   I  holds
for a system where the moment of inertia I about the 2. A source of sound is moving along a circular orbit of
axis of rotation is constant. Here the moment of inertia radius 3 m with an angular velocity of 10 rad/s. A
sound detector located far away from the source is
of the ‘rod-insect’ system changes with time.
executing simple harmonic motion along the line BD
But even if the moment of inertia about the axis of
 (see the figure) with an amplitude BC = CD = 6 meters.
 dL The frequency of oscillation of the detector is 5/ per
rotation changes, we can apply the equation   .
dt second. The source is at point A when the detector is at
Using this equation we get point B. If the source emits a continuous sound wave of
frequency 340 Hz find the maximum and the minimum

2 ♦ Science Pyramid ♦
frequencies recorded by the detector. Take the velocity As the source is far away from the observer, the
of sound in air as 330 m/s. velocities of the source and the observer can be taken
(IIT-JEE 1990, 7 Marks) along the same line, and the maximum and the
6m 6m minimum frequencies recorded can be calculated as
under : For the moments when the source is at E,
3m A S O 60 m/s
B C D
30 m/s E C

Solution: The answers, wherever the problem has been v  vo 330  60


min  0  340   225 Hz.
solved or given, are max = 442 Hz and min = 225 Hz. v  vs 330  30
These answers are not correct. The logic on the basis of
And when the source is at F,
which they arrive at these answers has flaws. In fact, O
S 60 m/s
this problem cannot be solved because a crucial fact has
F 30 m/s C
been neglected while framing the problem.
First, let us see how these answers have been v  vo 330  60
max  0  340  442 Hz.
calculated. v  vs 330  30
For source For observer Now, we will check whether the given information also
meets the requirements of an essential condition.
 = 10 rad/s v = 5/ per second.
Will the note emitted by the source while at point E
Radius of the circular Angular frequency  = (moving towards left with 30 m/s) necessarily be
orbit r = 3 m. 2 = 10 rad/s. received by the observer when it is at C (moving
Speed v = r = 30 m/s. Amplitude of SHM = 6m towards right with velocity 60 m/s)? After all the
distance between the source and the observer is long
Time period Maximum speed, at the enough… ‘the detector located far away from the
2  mean position a = 60 m/s. source'…; the wavelengths emitted by source when it is
T  s.
 5 at point E will reach the observer only after some time,
it will never reach the detector instantly. The time taken
by the sound to travel from the source to the detector
Let at time t = 0, there is no loss of generality in
can not be neglected. And this time need not be an
assuming so, the source be at point A.
30 m/s
integral multiple of /5 s!
E
The notes emitted by the source when it is at point E
60 m/s      2 
A  at t  20 , 20  5 , 20  5 ,...  will certainly be
B C D  
detected by the observer, but the position and velocity
F of the detector at the moment it detects these notes can
The source will be at E (with velocity 30 m/s towards not be ascertained; detector can be anywhere on the line
left) at the moments BD , and its velocity can have any value from 60 m/s
    2  3  4 towards left to 60 m/s towards right
t ,  ,  ,  ,  ,… (–60 m/s  v0  +60 m/s).
20 20 5 20 5 20 5 20 5
And at these moments the observer will be at C with Where did the examiner falter?
velocity 60 m/s towards right. He combined two problems in one.
E (1) A source moves along a small circle and a detector
60 m/s located far away on the line through the centre of the
A
circle and in the plane of the circle. (2) A detector
B C D
performs simple harmonic motion on a line and a
30 m/s
source located far away on that line. These two
F
problems can easily be solved. The moment of emission
The source will be at F (velocity 30 m/s towards right)
of a note and the moment of detection are not important
at the moments
here. But in the given problem they are. And the
examiner did not leave the choice of selecting these
3 3  3 2 3 3 moments of time to students. He clearly stated that the
t ,  ,  ,  ,…
20 20 5 20 5 20 5 source was at point A when the detector was at point
B. Probably that is where he faltered.
And at these moments the observer will be at C with
velocity 60 m/s towards left.

3 ♦ Science Pyramid ♦
3. A cylindrical block of length 0.4 m and area of cross- Let the temperature of the disc at time t be T. Let the
section 0.04 m2 is placed coaxially on a thin metal disc increment of the temperature of the disc in time dt be
of mass 0.4 kg and of same cross-section. The upper dT.
face of the cylinder is maintained at a constant 400 K
temperature of 400 K and the initial temperature of the l = 0.4 m
disc is 300 K. If the thermal conductivity of the A = 0 .04 m2
material of the cylinder is 10 Watt/m-K and the specific Cylinder K = 10 watt/m-K
heat of the material of the disc is 600 J/kg-K, how long
will it take for the temperature of the disc to increase to m = 0.4 kg
350 K? Assume, for the purposes of calculation, the c = 600 J/kg-K
thermal conductivity of the disc to be very high and the Disc T
system to be thermally insulated except for the upper Rate of heat flow through the cylinder = Rate of heat
face of the cylinder. (IIT-JEE 1992, 8 Marks) flow to the disc.
Solution: Here, apart from numerical values, you have KA(400  T ) dT
been given two conditions. or  mc
l dt
(i) The thermal conductivity of the disc is very high.
KA dT
(ii) The system is thermally insulated except for the or dt  m c
upper face of the cylinder. l 400  T
From the first condition you infer that the heating of the Integrating both sides with suitable limits,
disc is uniform, there is no temperature gradient across KA 350
the disc; the use of the phrase ‘the temperature of the or t   mc ln(400  T ) 300
l
disc’ is justified. The rate at which heat enters the disc,
is the rate of the temperature rise of the disc given by = mc (ln 50  ln 100)
dq dT KA
 mc , where m and c are the mass of the disc or t  mc ln 2
dt dt l
and the specific heat of the disc material respectively. lmc
or t ln 2.
From the second condition you infer that there is no KA
heat loss to the surrounding. All the heat that enters the Substituting the given numerical values,
top of the cylinder goes to the disc from its bottom.
0.4  0.4  600
But how can you account for the heat absorbed by the t ln 2  240 ln 2 s  166.3 s .
cylinder? After all, as the heat flow takes place, and as 10  0.04
the temperature of the disc increases, the temperature at
different cross-sections of the cylinder will also 4. An electron and a proton are moving on straight
increase. Don’t you think some heat would be absorbed parallel paths with same velocity. They enter a semi-
in raising the temperature at different cross-sections of infinite region of uniform magnetic field perpendicular
the cylinder? Of course, it would be. You can use the to the velocity. Which of the following statement(s)
dq kA(T ) is/are true?
heat flow equation  if at all the moments
dt l (A) They will never come out of the magnetic field
of time the heat flow is under steady state condition, the region.
heat entering the upper face of the cylinder is equal to (B) They will come out traveling along parallel paths.
the heat leaving its bottom face, no heat being absorbed (C) They will come out at the same time.
by the cylinder. Then how does the temperature at the (D) They will come out at different times.
different cross-sections of the cylinder rise?
(IIT-JEE 2011, 2 Marks)
So with the given conditions the problem can not be
Solution: In the key given on the official IIT-JEE
solved.
website the answer was (B, C) or (B, D) or (B, C, D).
You can solve the problem only if you assume that the This question has three answers! Strange isn’t it? How
specific heat of the material of cylinder is zero (or on the earth choices (C) and (D) can be true
negligible). That is, the temperature changes at different simultaneously?
cross-sections of the cylinder do not involve any
This question is very confusing. It can be easily shown
absorption of heat. With this condition, the problem can
that choice (B) is correct. Choice (C) may be correct,
be solved as follows.
may not be correct. Similarly choice (D) may be
correct, may not be correct. On the basis of the
information given in the question one cannot decide it.

4 ♦ Science Pyramid ♦
The particles enter a semi-infinite region of uniform     
magnetic field perpendicular to the velocity. We are 2π – 2α
    
given the angle the velocity vector makes with the
magnetic field, but not with the boundary of the region.     
The following figure shows three cases out of infinite   α   α 
A α α
ways the particles can enter the magnetic field. B
α α α α

p e
     
Let the proton and electron enter in parallel paths at an
      angle α as in the figure above. AB is the boundary
      separating the magnetic field region from non-magnetic
field region. The line AB extends to infinity.
p p e
p e e The path of proton subtends angle 2α at the centre of
the circle, while the path of electron subtends (2π -2α)
Note that the phrase “an electron and proton moving on at the centre of the circle. Do the required geometrical
straight parallel paths with same velocity.” does not calculations yourself.
necessarily mean that the two particles are side by side Hence time taken by the electron is
as shown in the figure below. (2  2 )me
te  , and that by the proton is
p
eB
2m p
tp  .
e eB
Now if te = tp, then
The two particles can be moving with same velocity on ( – α) me = α mp
parallel paths as shown in the figure below, and enter
the magnetic field at the same instant. me 
or   .
me  m p 1837
p
This is a very small angle. But conceptually this is a
possibility. And if the particles enter the magnetic field
e at this angle then the time taken by them to come out of
it will be equal. In that case choice (C) will be correct.
We know that a proton is 1836 times (approximately) In other situations choice (D) will be correct.
heavier than an electron. Once they enter the magnetic
It seems that the examiner assumed that the velocities
field, their angular velocities will be different due to the
of electron and proton were same, they were
difference in their masses. perpendicular to the magnetic field and also
eB perpendicular to the boundary, as shown in the figure
Angular velocity of proton  p  , and
mp below.
eB      
angular velocity of electron e  ,
me      
where B is the magnetic field, e is the magnitude of the      
charge on proton and electron, me mass of electron and
mp mass of proton.
Clearly, e   p . p e
But it was not written in the paper clearly. Paper did not
From this information can we really conclude that the
have any diagram also.
time taken by both the particles to come out of the
magnetic field will be different? Suppose the path taken
by electron is longer than that taken by proton. Then 5. A composite block is made of slabs A, B, C, D and E
there is a possibility that they come out at the same of different thermal conductivities (given in terms of a
time. Consider the following situation. constant K) and sizes (given in terms of length, L) as
shown in the figure. All slabs are of same width. Heat
‘Q’ flows only from left to right through the blocks.
Then in steady state

5 ♦ Science Pyramid ♦
0 1L 5L 6L Now since heat flows through the slabs only from left
heat
A B 3K E to right, you can also write heat flow equations as
1L
2K 4K 6K
1 2 q1 3K 3 4
C
3L 1 2K 2 q2 4K   6K 4
3
D 5K
4L
(A) heat flow through A and E slabs are same. 1 2 q3 5K 3 4
(B) heat flow through slab E is maximum.
(C) temperature difference across slab E is smallest. (2 K )( Lb)(1  2 ) (3K )( Lb)(2  3 )
(D) heat flow through C = heat flow through B + heat q1  
L 4L
flow through D. (IIT-JEE 2011, 3 Marks)
Solution: As per the official IIT-JEE site the answer of (6 K )( Lb)(3  4 )

this question is (A), (C), (D). But a rigorous analysis L
shows that the choice (C) is not correct, because under 3
the given conditions the behavior of slab E is pretty  2(1  2 )  (2  3 )  6(3  4 ) . …(2)
strange, and the phrase ‘temperature difference across 4
slab E ’ does not have any meaning. It also turns out (2 K )(2 Lb)(1  2 ) (4 K )(2 Lb)(2  3 )
that the choice (D) is also incorrect. q2  
L 4L
This question has many flaws. If you look at the
arrangement carefully, you can argue intuitively that the (6 K )(2 Lb)(3  4 )

behaviors of slabs A and E are strange. Thermal L
conductivity of slab D is more than that of slabs C and
B. Shouldn’t heat rush to take the path of least thermal  4(1  2 )  2(2  3 )  12(3  4 ) . …(3)
resistance? Shouldn’t heat flow through slab A from top And
to bottom, and through E from bottom to top too? To
visualize this, consider an unusual situation. Assume (2 K )( Lb)(1  2 ) (5 K )( Lb)(2  3 )
q3  
that the thermal conductivity of the slabs B and C is L 4L
zero. Heat flow occurs only through slab D. What will (6 K )( Lb)(3  4 )
be the pattern of heat flow through slabs A and E in that 
case? L
Anyway, let us go by what has been given in the 5
 2(1  2 )  (2  3 )  6(3  4 ) . …(4)
question. All the slabs allow heat flow only from left to 4
right. For lateral heat flow they offer infinite thermal Look at the equations (1) through (4) carefully. Without
resistance. Assume the temperature at the interfaces as solving them you can conclude that
shown in the figure below. If the thickness of each slab
is b, in steady state heat flow condition 2  2  2  2 and 3  3  3  3 .
heat
0 1L 5L 6L We can draw the following conclusions
A B 3K E 1. Slab A is made of 3 slabs.
1L
1 A1 2
2K C 4K 6K
3L 1 A2 2
D 5K
4L
1 2 3 4 1 A3 
2
(2 K )(4 Lb)(1  2 ) (3K )( Lb)(2  3 )
 2. Slab E is made of 3 slabs.
L 4L
3 E1 4
(4 K )(2 Lb)(2  3 ) (5K )( Lb)(2  3 )
 
4L 4L 3 E2 4
(6 K )(4 Lb)(3  4 )
 3 E3 4
L
 2(1 – 2) = (2 – 3) = 6(3 – 4). …(1)
If 1 and 4 are fixed, you can find 2 and 3 in terms of
1 and 4.

6 ♦ Science Pyramid ♦
3. Temperature difference across slab E has no 3K ( L  b) 3Kb
For slab B,   
meaning. It is (3  4 ) for some part, (3  4 ) for 4L 4
some other part and (3  4 ) for still other part. 4 K (2 L  b)
For slab C ,    2 Kb
4L
So choice (C) cannot have any meaning.
5 K ( L  b) 5 Kb
4. The temperature is not same everywhere at the right For slab D,   
end of slab A. Also the temperature is not same 4L 4
everywhere at the left end of slab E. Under these 3Kb 5 Kb
conditions, the formulation of equation (1) is not Clearly,   2 Kb.
4 4
correct.
 heat flow through slab B + heat flow through slab D
5. Heat flow through slabs A1  B  E1 = heat flow through C.
L 4L L 2 l
R1     The quantity is called thermal resistance. For a
(2 K )( Lb) (3K )( Lb) (6 K )( Lb) Kb KA
1  4 given heat flow rate, the temperature difference is
q1  . smallest if thermal resistance is smallest.
 2 
  1
 Kb  For slab A thermal resistance is  (α is some
8
Heat flow through slabs A2  C  E2
constant).
L 4L L 10 1
R2     For slabs (B + C + D) it is  .
(2 K )(2 Lb) (4 K )(2 Lb) (6 K )(2 Lb) 12 Kb 4
1  4 
q2  . For slab E it is .
 10  24
 
 12 Kb  For slab E the thermal resistance, hence the temperature
difference across it, is the smallest.
Heat flow through slabs A3  D  E3
L 4L L 22 6. Four point charges, each of +q, are rigidly fixed at
R3    
(2 K )( Lb) (5 K )( Lb) (6 K )( Lb) 15Kb the four corners of a square planar soap film of side ‘a’.
The surface tension of the soap film is γ. The system of
1  4 charges and planar film are in equilibrium, and
q3  .
 22  1/ N
   q2 
 15Kb  ak  , where ‘k’ is a constant. Then N is
  
Clearly, q1  q3  q2 . (IIT-JEE-2011, 4 Marks)
Hence choice (D) is not correct. Solution: This question defies every conceivable logic.
In the steady state, the rate of heat flow through any There is a fundamental flaw in framing this problem.
cross section is same  choice (A) is correct. It has been given in the question that the point charges
We have, rate of heat flow are rigidly fixed at the corners of a square planar soap
film. How? How do you rigidly fix a charged particle
dQ KA(2  1 )
 , …(1) to a liquid film.
dt l Anyway, let us go by what examiner says and assume
where K is thermal conductivity of the material, A is that somehow it has been done. The square planar soap
area of cross section of slab and l the length, θ 2 – θ1 is film of side a with point charge +q rigidly fixed at each
temperature difference across the ends of slab. of its corners is in equilibrium. See the figure below.
Now for the slabs B, C and D the temperature a
difference is same. q q

Hence the rates of heat flow through these slabs depend


a a
KA
on the factor .
l q q
Let us represent this factor by  . a

7 ♦ Science Pyramid ♦
Cut the film along the dotted line shown. Draw the F
forces on the point charge and the small part of the soap a/2
film with it. F
F1
a a/2

2l

a
Fa. Fine!
The electrostatic repulsion F1 on the charge has a finite
F
value. The surface tension force on the small part of the
film is infinitesimally small. We have the liberty to 3a/4
F
make l as small as we please. Can this part of the
system ever be in equilibrium?. How can the net force a a/2
on this system be zero?
Now consider another system: the four charges and a
very thin thread of the soap film. Forces on this system 3a
are shown in the figure. The columbic force on each 2
charge has been shown in two components.
F F
Fa, and F 
3a
2
too ?
Can you now find the mistake in the framing of the
F F problem? Possibly yes.
q q
Now let us solve the problem the way examiner wanted
us to do it.
q q We take the line AB and equate the net vertical force on
F F AB to zero.
F A B
F q q
Total surface tension force on each side = 2 a.
One can say that if 2F = 2a, the above system is in
equilibrium.
q q
Surely, it is. Don’t you see an elephant hanging from a D C
thin cotton thread in this system, tension in the thread
F1
being equal to the weight of the elephant! F1
F2
Consider another system of two point charges and a F2
q
thread of the soap film. F3 F3
A Bq
F F 2γa
F
F F 2 2
q k q
F1  2

2a 4 0 a a2
One can say that if 2F = 2a net force on the system is
zero, it is in equilibrium. But what about the forces q2 k q 2
F2  2

which act along the thread of the soap film? Will they 4 0 ( 2a ) 2a 2
not break it apart?

The following system is still a better choice: Hence, 2 F1  2 F2 cos  2 a
4
F
a/2 2k q 2 2k q 2
2l F or   2 a
a2 2a 2
a
 1 
F = a, and F = 2l too? q2 k  1   3 2
 2 2 k q
or a3   ,
You may be tempted to consider a finite part of the  
film. Two such systems are given below.

8 ♦ Science Pyramid ♦
 1   1  2
where k 3  k  1   1  q
 2 2 3  2 2 k 3q2
or x  
2 1/3
40  
q
 a  k    N  3. 1/3
 q2 
    xk  .
A smart idea: If you read the problem carefully, you   
can see that the value of N can be found out without Since the equilibrium is attained at x  a,
doing any calculations. The problem has been worded 1
in such a way that you can arrive at the answer without  q2  3
calculating the coulomb force on the charges. From a  k   . Hence N = 3.
Coulomb’s law and principle of superposition you can   
write that the Coulomb’s force on a charge, A smart idea again: You can arrive at the answer
q2 smartly without doing any calculations. Write the total
Fc = (a constant) . mechanical energy assuming that the side of the square
a2 is x as
2k 3 q 2 q
Just write this constant as 2k3. Then  2 a , E C  2 x2 
a2 x
1/3
 q2  Minimize E for x. Let the equilibrium be attained at
which gives a  k   . x = a. Replace the constant term by k intelligently.
  
I am sure that many students have thought of this and
7. A boy is pushing a ring of mass 2 kg and radius
have done this problem in exactly this way in the
0.5 m with a stick as shown in the figure. The stick
examination hall. Many brilliant students might have
applies a force of 2 N on the ring and rolls it without
wasted precious exam time in figuring out the problem.
slipping with an acceleration of 0.3 m/s2. The
Those who are in the habit of doing things correctly
coefficient of friction between the ground and the ring
may have been victims of great confusion and
is large enough that rolling always occurs and the
perplexity.
coefficient of friction between the stick and the ring is
Second Solution: Let x be the side of the square as (P/10). The value of P is
shown in the figure below. Stick
q x q
A B

x x

q q
D x C Ground
For the equilibrium condition the electrostatic potential (IIT-JEE 2011, 4 Marks)
energy + surface energy must be a minimum. Let the
equilibrium be attained at x = a. Solution: As per official IIT-JEE website answer of
Now, the total energy, this question is P = 4. It was written explicitly in the
paper that answer is a single digit integer ranging from
q2 q2 0 to 9. Students faced great difficulty while solving this
E  4  2   (2 x 2 )
40 x 40 2 x problem.
‘The stick applies a force of 2 N on the ring and rolls it
(4  2) q 2 without slipping with an acceleration of 0.3 m/s2’ . This
  2 x 2 .
4 0 x statement is not entirely incorrect, but it is certainly
d (E) confusing. What does the examiner mean by ‘the stick
For equilibrium  0, applies a force of 2 N on the ring’? Is it the normal
dx force on the ring by the stick or the total force on the
(4  2)  q 2  ring by the stick? In the first sentence of the question it
or     4 x  0 is given that the boy is pushing the ring. On this basis it
40  x 2 
can be said that 2 N is the normal force acting on the
ring by the stick, not the total force.

9 ♦ Science Pyramid ♦
Let the coefficient of friction between the stick and the Since the ring rolls on the ground, its angular
ring be . The forces acting on the ring are:  0.3 3
acceleration   cm   radian/s2.
1. 2-N normal force by the stick. R 0.5 5
2. Frictional force of value 2N vertically downward. Now the force and torque equations are
Note that this frictional force is kinetic in nature. As the N – f = m acm = 2  
ring rolls on the ground, it rotates about its centre of
mass in anticlockwise sense. The point of contact of the f R – NR =Icm 
ring with stick is in motion. The frictional force is 3
or f  0.5 – N  0.5 = 2 (0.5)2  
opposite to the velocity of point of contact. 5
3. Weight of the ring 2  10 N vertically downward or f – N = 0.6 …(2)
acting at the centre of gravity. From equations (1) and (2),
4. Normal force N1 at the point of contact by the
1.2
ground. N .
5. Frictional force f at the point of contact with the 1 
ground. This friction is static in nature. Since the ring Now if the total force on the ring by the stick is 2 N,
always rolls on the ground, the frictional force on the
N 2  (N ) 2  2
ring by the ground is static friction less than its limiting
value. The direction of this friction is towards the right. 2 2
 1.2   1.2 
Why? The tendency of motion of the point of contact is or     2
towards left. This is due to the action of 2-N force.  1    1   
The free body diagram of the ring is as shown in the or  = 0.361, 2.763.
figure below.
α  P = 3.61, 27.63, which is not a SINGLE-DIGIT
INTEGER.
acm = 0.3 m/s2 2N
20 N
Many students wasted lots of precious exam time on
Radius = R
2 N this problem. They took the 2 N force as the total force
f on the ring by the stick.
N1
8. A thin ring of mass 2 kg and radius 0.5 m is rolling
The force and torque equations for the ring are
without slipping on a horizontal plane with velocity 1
2 – f = 2  a cm = 2  0.3 …(1) m/s. A small ball of mass 0.1 kg, moving with velocity
N1 – 20 –2 = 0 …(2) 20 m/s in the opposite direction, hits the ring at a height
2
f  0.5 – 2  0.5 = 2  (0.5)   …(3) of 0.75 m and goes vertically up with velocity 10 m/s.
The constraint equation is Immediately after the collision,
acm = R  or 0.3 = 0.5   …(4) 10 m/s
From these equations 20 m/s

 = 0.4
P 0.75 m 1 m/s
So,  4.0 or P  4.
10
But one can also read the problem in a different way.
One can assume that 2-N force is the total force acting (A) the ring has pure rotation about its stationary CM.
on the ring by the stick. In that situation the free body (B) the ring comes to a complete stop.
diagram and force equation will be as under.
(C) friction between the ring and the ground is to the left.
α
(D) there is no friction between the ring and the ground.
acm = 0.3m/s2 20N N
(IIT-JEE 2011 3 Marks)
Radius = R
N Solution: The answer is A or A,C as per official IIT-
f JEE key. The question created a lot of confusion and
N1 stir in the country. The examiner took great care in
Radius of the ring R = 0.5 m. choosing the values of masses, velocities and lengths.
He faltered in phrasing choice (C). Probably, to make
Acceleration of center of mass, acm = 0.3 m/s2. up for his misdoings, IIT-JEE organizing body came

10 ♦ Science Pyramid ♦
with two answers: A or AC. But a rigorous analysis 1Ns
shows that even choice (A) cannot be correct.
The examiner did not consider the frictional force on 2 Ns 0.25 m
the ring by the ground during the collision. The values 0.5 m
of masses and velocities and the answer given on IIT- mg
JEE site corroborate this fact. While solving this
problem if you do not consider the frictional force on N
the ring by the ground then you can show that just after
the collision the velocity of centre of mass of the ring is 3
zero; but the ring has an angular velocity about the 1  0.5   2  0.25  2  (0.5) 2 (  2)
2
centre of mass and the frictional force on the ring by the
or  = 1.866 rad/s anticlockwise.
ground is toward left. But the examiner somewhat mis-
worded it: the friction between the ring and ground is to That is, immediately after collision, vcm = 0 and
the left.  = 1.866 rad/s anticlockwise. The ring rotates about
stationary centre of mass. Velocity of the point of
In realty, a frictional force, impulsive in nature, that is,
contact with ground is to the right. The friction on the
of a relatively large value, acts on the ring during the
ring by the ground is to the left.
collision. Whether this kind of frictional force is to be
considered or not created a great deal of confusion. You can also arrive at these answers by conserving
linear momentum, and angular momentum of the ring
To begin with, we consider the examiner’s point of
ball system relative to the ground.
view. First analyze the motion of the ball. Suppose the
10 m/s
forces acting on the ball are as shown in the figure. 2 rad/s
20 m/s 
10 m/s
0.1 kg 20 m/s F1 kg 0.1 kg
F2 0.1 kg
Before collision After collision 1 m/s
Forces on ball 2 kg vcm
during collision

If the time of collision is t, Just before collision Just after collision

F1t = 0.1  20 = 2 Ns …(1) Conservation of linear momentum


F2t = 0.1  10 = 1 Ns …(2) –0.1  20 + 2  1 = 2 vcm  vcm = 0.
(Please take care of signs and directions.) Conservation of angular momentum relative to the
Next consider the ring. Ignoring the frictional force ground
during the collision, the forces on the ring are as shown  1
 0.5  (0.5) 2  2  0.1
2
   2  

20  0.75
 
in the figure: M vcm r Icm  mvi r
F2 1
2 rad/s 
F1 3
 2  (0.5)2   0.1 10  0.5 
 2
Icm    
2 mv
1 m/s f r
mg vcm
3
=1+ = 1.866 rad/s.
N 2
Before collision During collision After collision
Now let us check whether the force on the ball by the
ring is normal to the ring or not. The situation is shown
Linear impulse = change in linear momentum gives in the figure below.
–F1 t = 2 vcm – 2  1 F2
or –2 = 2 vcm –2 (From (1)) F

or vcm = 0.
F1 0.25 m
And angular impulse about CM = change in angular
momentum about CM gives, 0.5 m

From the above figure,

11 ♦ Science Pyramid ♦
F2 1
tan   . From equations (1) and (2)
F1 2 For mastering the concepts,
 1 1  problem solving techniques and
  tan   , tan 30  and 2  3 
 2 3  for all possible shortcuts, read
Hence the total force on the ball is not normal to the
ring. Further let us resolve the resultant force into
components: 1. The Art of Problem Solving in Physics
(1) Normal component F = F cos (30 – ), the Volume-1
component which is normal to the ring.
(2) Tangential component F = F sin (30 – ), the By SP Neelam
component which is tangential to the ring. (Price : Rs : 600/-)
These two components and their reactions are shown in
the figures below.  Kinematics
F  The Fundamental Equation of Dynamics)
F F  Law of Conservation of Energy, Momentum and
Angular Momentum)
 Universal Gravitation
F  Dynamics of a Solid Body
 Elastic Deformation of a Solid Body
 Hydrodynamics
 Equation of the Gas State Processes
Force on ball by ring Force on ring by ball  The First Law of Thermodynamics Heat Capacity
ring
From the above figure it can be inferred that during the  Kinetic Theory of Gases
collision there is a frictional force F = F sin (30 – )  Liquid Capillary Effect
on the ball by the ring. The reaction of this force acts on  Heat Conduction
the ring. If the ball applies a frictional force on the ring
during the collision, so does the ground. It can be 2. The Art of Problem Solving in Physics
shown that the point of contact of the ring has a Volume-2
tendency of motion relative to the ground.
Hence one may expect a frictional force on the ring by
By SP Neelam
the ground. Moreover, during the collision, the normal
force on the ring by the ground takes a large value. This
(Price : Rs : 650/-)
can be inferred from the free body diagram of the ring.
 Constant Electric Field in Vacuum
10 m/s  Conductors and Dielectrics in an Electric Field
20 m/s 2 rad/s   Electric Capacitance, Energy of an Electric Field
 Electric Current
0.1 kg 2 kg  Constant Magnetic Field, Magnetics
 Electromagnetic Induction
1 m/s  Motion of Charged Particles in Electric and
vcm
Magnetic Fields
 Mechanical Oscillations
Before collision After collision  Electric Oscillation
 Elastic Waves, Acoustics
From conservation of linear momentum,  Geometrical Optics
2  1  0.5 + 2  (0.5)2  2 – 1.0  20 – 0.75  Interference of Light
 Scattering of Particles Rutherford-Bohr Atom
3
= 2  vcm    Radioactivity
2  Nuclear Reaction
From this single equation you cannot find out two
unknown quantities vcm and  Therefore, you cannot
really conclude anything.
So the most important point is whether to consider the You know you are good.
impulsive friction or not. The examiner (probably) did These books will help you prove it.
not consider it.

12 ♦ Science Pyramid ♦
Random Problems from ‘The Art of problem Solving in Physics’ Volume : I

1. A cannon fires successively two shells with velocity 2v0 sin(1  2 )


 .
; v0  250 m/s , the first at the angle 1  60 and the g cos 1  cos 2
second at the angle 2  45 to the horizontal, the Second Solution: Let the Shell-1 fly for time t1 and
azimuth being the same. Neglecting the air drag, find Shell-2 for time t2 before colliding at point A, as
the time interval between firings leading to the collision
shown in the figure. During these times t1 and t2 , the
of the shells
Solution: Let Shell-1 be projected from origin with shells undergo same horizontal and vertical
displacements. That is,
velocity v0 at angle 1 to the horizontal at moment
v0 cos 1  t1  v0 cos 2  t2 …(1)
t  0, and Shell-2 with same velocity v0 at angle 2 at t1
y
the moment t   . Let them collide at moment t  t at
point ( x, y) . Shell-1, clearly, flies for time t and Shell- v0 t2 A

2 for time (t  ) before they collide. v0

Motion along x-axis:


O x
Shell-1: x  v0 cos 1  t …(1)
1 1
Shell-2: x  v0 cos 2  (t  ) …(2) and v0 sin 1  t1  gt12  v0 sin 2  t2  gt22 …(2)
Motion along y-axis: 2 2
On dividing equation (2) by equation (1), you get,
1
Shell-1: y  v0 sin 1  t  gt 2 …(3) 1 1
2 v0 sin 1  gt1 v0 sin 2  gt2
2  2 ,
1
Shell-2: y  v0 sin 2  (t  )  g (t  ) 2 …(4) v0 cos 1 v0 cos 2
2 which gives
y
2v0
v0 (x, y), t = t sin(1  2 )  t1 cos 2  t2 cos 1 …(3)
Shell 1 g
v0
On solving equations (1) and (3), you get
t=0 1 2 2v cos 2
O x t1  0  sin(1  2 )  .
Shell 2
t=
g cos 2  cos2 1
2

From equations (1) and (2) cos 1


Required t  t1  t2  t1  t1
v0 cos 1  t  v0 cos 2  (t  ) cos 2
cos 2   cos 2  cos 1
or t . …(5)  t1 
cos 2  cos 1 cos 2
And from equations (3) and (4), 2v0 cos 2 cos 2  cos 1
1 1  sin(1  2 )  2 2

v0 sin 1  t  gt 2  v0 sin 2  (t  )  g (t  ) 2 …(6) g cos 2  cos 1 cos 2
2 2
2v0 sin(1  2 )
cos 2    .
Substituting t  from equation (5) in g cos 1  cos 2
cos 2  cos 1
equation (6), Third Solution: Shell-1 is thrown at moment t  0,
2 with velocity v0 at angle 1 to the horizontal. And at
 cos 2    1  cos 2   
v0 sin 1   g  moment t  , Shell-2 is thrown from the same point
 cos 2  cos 1  2  cos 2  cos 1  with velocity v0 at angle 2 to the horizontal. Refer to
2
 cos 2    1  cos  2    the coordinate system given in the figure below.
 v0 sin 2      g    .
 cos 2  cos 1  2  cos 2  cos 1 
Solve this equation for .

1
v0 sin 1  g  Solution: (a) The motion of the particle is depicted in
the figure below.
y v0 cos 1
t=
Since l is displacement from the initial position
measured along the arc, velocity v is given by
v2
dl
v  a cos t , and tangential acceleration is given
v0 sin 2 dt
2
O x dv
t= v0 cos 2 by a   a2 sin t.
dt
At the moment t   , the Shell-1 is at point The radial acceleration
 1 2 v 2 ( a cos t ) 2 l
 v0 cos 1  , v0 sin 1    g   and has x- and y- ar  
 2  R R R
components of velocity as v0 cos 1 and v0 sin 1  g  a 2 2 cos 2 t O

respectively. Figure below depicts the positions and R
velocities of Shell-1 and Shell-2 at moment t  . Now total acceleration
v0 sin 1  g 
aTotal  a 2  ar 2
2
v0 cos 1  a 22 cos 2 t 
1 2
 a 2 4 sin 2 t   
 R
 v0 cos 1  , v0 sin 1    g    
 2 
a 2 2
For l  0, a sin t  0 or t  0 , and so aTotal  .
R
v0
v0 sin 2 For l  a, t   2 , and for this value of t ,

(0, 0)
2 aTotal  a2 .
2 2v0 cos 2 2
Let the game start at the moment t  . This moment  a 2 2 cos 2 t  2 4 2
(b) aTotal     a  sin t …(1)
onwards acceleration of Shell-2 with respect to Shell-1  R 
is zero. How? If you look at Shell-2 from Shell-1, its a 44 cos 4 t
velocity must be aimed at Shell-1 for the shells to Let f   a 2 4 sin 2 t .
R2
collide. That is, the direction of velocity of Shell-2 wrt
  df
Shell-1 v21 must be in the direction of vector 21. From  0 gives
dt
the figure, a 4 4
   4cos 3 t sin t    a 24  2sin  t cos t    0
v2 1  (v0 cos 2  v0 cos 1 )i  (v0 sin 2  v0 sin 1  g ) j
R2
  1 
and 21  (v0 cos 1  )i   v0 sin 1    g  2  j R2
or cos 2 t  .
 2  2a 2
 
Since the vector v2 1 is in the direction of vector 21 , For the values of t given by the above equation aTotal
will take its minimum value (Why?). On putting
v0 cos 2  v0 cos 1 v0 sin 2  v0 sin 1  g 
 , R2
v0 cos 1   1 cos 2 t  in equation (1),
v0 sin 1    g 2 2a 2
2
2
2v0 sin(1  2 )  R
which on solving for  gives    . (aTotal ) min  a2 1    .
g cos 1  cos 2  2a 
Second Solution:
2. A particle moves along an arc of a circle of radius R
2
according to the law l = a sin t, where l is the  a 2 2 cos 2 t  2 2
aTotal     (a sin t )
displacement from the initial position measured along  R 
the arc, and a and  are constants. Find:
2
(a) the magnitude of the total acceleration of the a
 a2 2
cos 4 t  (1  cos 2 t )
particle at the points l = 0 and l = ± a; R
(b) the minimum value of the total acceleration
( aTotal ) min and the corresponding displacement lm.

2
  a2 on angle  if the coefficient of friction  = tan  and at the
 a2  R2
 a2 1    2 cos 2 t  2
1  R 2 cos t   a 2
initial moment 0 = /2.
 R
   Solution: How to (rather from where to) get started? This is
y

 the only problem in this question. Even after having drawn
It can be seen from the above equation that aTotal will the free-body diagram correctly you will not be sure of how to
 
take its minimum value when y is maximum. apply F  ma to the block, how to set the coordinate
1. From AM  GM , system!
The forces on block A sliding on the incline are-
 a2   a2 
2 2
 2 cos t    1  2 cos t  (i) The weight mg acting vertically downward. Shown
R   R  y in the figure are two components of the mg  mg sin 
2
down the incline, along the line of maximum slope and
1
or y  mg cos , normal to the incline shown as mg cos   .
4
1
(ii) The normal force, N , normal to the incline, shown
 ymax  . as N  . Note that N  mg cos . The block moves
4
2 along the incline. It does not have any acceleration
1 R2  R normal to the incline.
Then (aTotal ) min  a2 1   a2 1    .
4 a2  2a  (iii) The frictional force f k   k N   k mg cos . This
2. The term y is maximum when force is parallel to the inclined plank in opposite
a2 a2 direction of velocity of the block. Velocity of the block
cos 2 t  1  2 cos 2 t. Why?
R 2
R is along the incline tangent to the path it follows on it.
c At moment t shown in the figure, velocity of the block
( p  q  c, then p  q is maximum when p  q  . )
2 makes angle  with the line of maximum slope at that
 a2   a2  point.
Since  cos 2 t   1  2 cos 2 t   1,
2
R   R  N fk
a 2   a 2 
2 2 mg cos   A
 2 cos t    1  2 cos t  will be maximum when 
 R   R  v k
2 2

a a  1 mg sin 
cos 2 t  1  2 cos 2 t    . ˆ 
R2 R  2
Therefore, fk = N
2 2 = (mg cos )
1 1 R  R  x
(aTotal ) min  a 2 1    2  a2 1    .
2 2 a  2a  You have to find v in terms of . Should you form a
differential equation in terms of v and ? No. Why?
Third Solution: You try to do it to get the answer of “why”. Should you
 a 22 cos 2 t  2 2 try to relate a, a x and  ? Many of you will do the
aTotal     (a sin t )
 R  mistake of writing that ax = a cos . Why is ax =
a cos  not correct? If you can answer this question
a 2 2 cos 2 t without any calculation whatsoever your fundamentals
  a 2 4 (1  cos 2 t )
R dvx
a2 are OK. What next? Clearly vx = v cos , and =a x,
 a 2 cos 4 t  R 2 cos 2 t  R 2 . dt
R
dv
The quadratic expression in cos 2 t under the square = a . So try to relate a x and a. But how to do it?
dt
root sign will take its minimum value when
Just write force equations along the directions ˆ and x̂ .
R2
cos 2 t  . From F = ma.
2a 2 mg sin  cos   mg sin  = ma  … (1)
(If a > 0, then the expression ax 2  bx  c takes its And from Fx = max,
b mg sin   mg sin  cos  = ma x … (2)
minimum value when x   .)
2a From equations (1) and (2),
3. A small block A is placed on an inclined plane forming an a = a x … (3) [So simple!]
angle  with the horizontal, see the figure and is imparted an dv dv
initial velocity v0. Find how the velocity of the block depends or  x or dv =  dvx
dt dt

3
 1 1 
or  dv   dvx or v = vx + c … (4)
p3  m 2 gh  3  2  .
Now when v = v0, vx = 0. [ initially  = /2, v0 is   
Total momentum delivered to the slab by the ball
perpendicular to x-axis]
or v0 = 0 + c p   p1  p2  p3  ...
or c = v0 1   1 1  1 1 
 m 2 gh   1  m 2 gh  2    m 2 gh  3  2   ...
Therefore, v =  vx + v0           
or v = v cos  + v0 [ vx = v cos ]  2 2 2    1 
 m 2 gh  1   2  3  ...   m 2 gh  
v0        1 
or v = .
1  cos  50  1.25  1 
4. A steel ball of mass m  50 g falls from the height
  2  9.8 1.0    0.2 kg m/s.
1000  1.25  1 
h  1.0 m on the horizontal surface of a massive slab. Second Solution: The ball hits the slab for the first
Find the cumulative momentum that the ball imparts to 2h
the slab after numerous bounces, if every impact time with velocity 2gh after time t1  . Then it
g
decreases the velocity of the ball   1.25 times.
2 gh
Solution: The process is depicted in the figure below. rebounds with velocity , goes up for time t2 and

then comes down in time t2 to hit the slab for the
1 2h
second time, where t2  . After the second
h  g
1
impact the ball bounces with speed 2 gh , it goes
h 2
1

2gh 1
2gh
1
2gh up for time t3 and then comes down in time t3 to hit
2 3
1 2h
the slab for the third time, here t3  .
1 1 2 g
2gh 2gh 2gh
 2 The ball is in air for a total time
I Impact II Impact III Impact
t  t1  2t2  2t3  ... ad infinitum
Change in momentum of the ball during first impact 2h 1 2h 1 2h
1  2 2 2  ...
p1   m 2 gh  ( m 2 gh ) . g  g  g

2h    1 
Here, the downward direction has been taken as   .
positive. g   1 
The momentum delivered to the slab during first impact During this time t , the gravity force imparts a total
1 1  momentum mgt to the ball. Eventually the ball comes
p1  m 2 gh  m 2 gh  m 2 gh   1 .
   to rest. So it delivers to the slab all the momentum it
Change in the momentum of the ball during second acquires during its motion.
Therefore, the cumulative linear momentum that ball
impact
1  1    1
imparts to the slab  mg t  m 2 gh  .
p2  m 2 2 gh   m 2 gh  .   1 
   
The momentum delivered to the slab during second
impact
1 1  1 1
p2  m 2
2 gh  m 2 gh  m 2 gh  2  
    
Similarly, the momentum delivered to the slab during
third impact

4
Random Problems from ‘The Art of problem Solving in Physics’ Volume : II

1. A very long uniformly charged thread oriented along a thin strip of radius r and thickness dr is
the axis of a circle of radius R rests on its centre with 1 
one of the ends. The charge of the thread per unit length  2r dr. Whence the total flux through the
 40 r
is equal to  . Find the flux of the vector E across the
circle,
circle area. R
Solution: The expression for the electric field at a point 1  R
   2r dr  .
at a distance r on the perpendicular passing through 0
40 r 20
one of the ends of a long thread (calculated in Problem
Second Solution: The solid angle subtended by a right
3.14) can be used to find the flux through the circle in a
circular cone on its vertex is  = 2 (1 – cos ), where  is
very simple way. half of the cone angle. This proposition can be easily
1  established using the definition of solid angle.
40 r Electric flux through an area dS due to a point charge q is
related to the solid angle the area dS subtends at the charge
as follows.
  q dS cosθ q  .
1  d  E  d S  
4πε 0 r2 4πε 0
40 r
r where d is solid angle subtended by area dS on the point
 charge.
O
Consider an element of length dx of the thread at a
distance x from centre of circle.
R The charge on the element of length dx is dq = dx.
The solid angle subtended by the circle on dq is
 = 2 (1 – cos ).
The component of the electric field which is parallel to  
the thread is normal to the circle and the component x
Ω  2π  1  .
 
which is perpendicular to the thread is parallel to the  R2  x2 
circle. Are you comfortable in visualizing this in three The electric flux passing through the circle due to
dimensional space? element charge dq
1  Ωdq 2π  x 
d   1   λdx
dr
40 r 4πε 0 4πε0  
R2  x 2 
Circle Flux through the circle due to the thread of length l ,
λ l  x 
Thread  0 1   dx
1  2ε 0  
  R  x2
2

40 r
λ
R
r

2ε 0  l
l 2

 R2  R .

R2
If the thread is long,  0, whence
l2
The thread is below the circle λl  R2 R 
passing though its centre  1  1   
Which component of the electric field contributes to 2ε 0 
 l2 l 

flux? The one that is perpendicular to the circle or the λR
one parallel to it? How does the component of electric  .
2ε0
field that is normal to the circle’s plane vary with the
distance of the point from its centre? Now, finally can Third Solution: Nobody remembers the expression for
you exploit symmetry in computing the integral the field calculated in Problem 3.14. Most of you may
  not be able to apply the idea of solid angle to solve this
 E  d s ? From symmetry considerations, flux through

1
problem. Suppose, all you remember is the field of a   R2 
1 q  lim    R
point charge, and definition of flux 
20 t   t  t  R
2 2 
40 r 2 
  
d   E  dS . If that is the case, how would you solve  (0  R)
2 0
the problem? It’s easy. Consider a point charge on the
thread. Find the flux through the circle as if this were R
 .
the only charge present. But, of course, this is not the 2 0
only charge! So integrate all over the thread. We have used the most basic manipulations of calculus
dE sin  to arrive at the answer.
2. A ball of radius R is uniformly charged with the volume
dE density  . Find the flux of the electric field strength vector
dE cos 
across the ball's section formed by the plane located at a
r
distance r0 < R from the centre of the ball.
 Solution: This is an absolutely simple problem. All you
x
dx need to do is to calculate the charge contained in the
R
cone whose vertex is at the centre of the sphere and
whose base is the ball’s section formed by the plane
located at a distance r0  R from the centre of the ball.
Refer to the figure above. Flux through a thin strip of r 2 h
Apply the formula V  , to obtain the volume of
radius r and thickness dr if only dq  dx were 3
present is dE cos   2rdr. Integration over the circle the cone described above. You have,
gives the flux through it if only charge dq were ( R 2  r02 )  r0
V . [ h  r0 ]
present. 3
R
1 dx x
d   2 2
 2rdr. R 2  r02
0
4 0 (x  r ) x  r2
2
R
[Note that x is the variable of integration for thread and
not for the circle.]
The contribution of the whole thread to the flux through r0
the circle is
R
1 xdx
   40 ( x 2  r 2 )3/2 2rdr.
0 0
First integrate over the circle, then over the thread. Thus, charge contained in the cone,
R 1
rdr 1 x  q    ( R 2  r02 ) r0 .
 ( x 2  r 2 )3/2  x 1  x 2  R2  . 3
0   Whence the required flux,
The integration is done by assuming x 2  r 2  t and 1
2rdr  dt , here x is a constant. Now substitute q 
   R 2  r02 r0 
  3
dt 0 0
rdr  and R( x 2  r 2 )  t 3/ 2 . Whence
2 
 r0 ( R 2  r02 ).
 30
x 1  x 
   1   dx Aren’t you amazed? And you know, most of the
2  x  2 2 
0 0  R x 
problems you are asked to solve are this simple!
  t  xdx   Second Solution: Consider a thin strip element of
 lim  t    
20 t   0  R 2  x2   radius y  0  r  R 2  r02  and thickness dy. Draw a
 

 
lim t  t 2  R 2  R
2 0 t   picture in the mind as visualization of the situation is
the most important aspect of problem solving. As
computed in Problem 3.16(b), electric field at each

2
 
 ρr  B
point of the ring is E  where r is the radius
3ε 0
vector of the point on the strip from the centre of the  
 r r
sphere. Note that r is a variable vector. 
j  O  O
The electric flux through this circular strip is r l r
ρrds cos θ
d 
3ε 0
2πρr y dy
 .
3ε 0  1  
B  0 [ j  r ].
Total flux through the circular surface 2

R 2  r02 2πρr0 Using now this formula we can write expression for B1
 y dy 
0 3ε 0 and B2 ,
πρr0 ( R 2  r02 )   
 . B1  0 [ ˆj  r ]
3ε 0 2
   
3. Inside a long straight uniform wire of round cross- and B2  0 [ j  r ].
section there is a long round cylindrical cavity whose 2
 
axis is parallel to the axis of the wire and displaced On substituting the expressions for B1 and B2 in
from the latter by a distance l . A direct current of equation (1) you get

density j flows along the wire. Find the magnetic          
B0  0 [ j  r ]  0 [ j  r ]  0 [ j  (r  r )].
induction inside the cavity. Consider, in particular, the 2 2 2
case l  0.      
Figure shows that r  l  r , whence r  r   l , and
Solution. In accordance  1  
with the principle of B  0 [ j  l ].
2
superposition, the 
required quantity can be Thus, in our case the induction B of the magnetic field
represented as follows: in the cavity is uniform, and if the current is flowing
   
B0  B1  B2 , …(1) towards us see the figure, the field B lies in the plane
 l
of the figure and is directed as shown.
where B1 is the magnetic Additionally, for the sake of clarity and practice we
induction of the compute the magnetic induction at a few more points.
conductor without cavity, Let the current density ĵ

while B2 is the magnetic be in the sense B1
induction of the field at represented by
B2 = 0
the same point due to the  , coming out of the j
current flowing through page. O O
the part of the conductor (i) At point on the axis of
which has been removed in order to create the cavity. cavity
Thus, the problem requires first of all the calculation of B1  2l  0 j R 2

magnetic induction B1 inside the solid conductor at a B2  0
distance r from its axis. Using the theorem on
B  B1  B2 .
circulation, we can write 2rB  0 r 2 j , whence
(ii) At the axis of the
1 wire:
B   0 rj. This expression can be represented with the
2 B1  0
help of the figure in the vector form:
B2  2l   0 j r 2
B  0  B2 B2

Can you tell the


magnitude and direction
of B ?

3
(iii) At point 1 in the (ix) At point 7 in the  7
B1
figure: B1 figure:

B1  2R   0 j R 2 B2 B1  2R   0 j R 2 B2

B2  2( R  l )   0 j r 2 1
B2  2 R 2  l 2  0 j r 2
B  B1  B2 .   
B  B1  B2 O'

Vector B2 is
perpendicular to line
(iv) At point 2 in the B1 O7.
figure:
B2 4. A beam of non-relativistic charged particles moves
B1  2(l  r )   0 j (l  r ) 2
without deviation through the region of space A, see the
B2  2r   0 j r 2 figure, where there are transverse mutually
2
B  B1  B2 . perpendicular electric and magnetic fields with strength
E and induction B. When the magnetic field is switched
off, the trace of the beam on the screen S shifts by x .
Knowing the distances a and b, find the specific charge
(v) At point 3 in the q/m of the particles.
figure: S
2 B1
B1  2(l  r )   0 j(l  r )
B2  2r   0 j r 2
   3 A
B  B1  B2 x
Here, B  B1  B2 B2

(Algebraically)

a b
(vi) At point 4 in the
figure: Solution: This problem is essentially the experimental
setup devised by J J Thomson for determining the
B1  2l   0 j l 2
4 specific charge (e/m) of an electron, a fundamental
B2  2(2l )  0 j r 2 particle whose discovery he is credited for. First, we
B2
B  B1  B2 shall understand the basic physics involved in the
B1 experiment and computation.
Suppose that a beam consisting of particles of charge q
(vii) At point 5 in the and mass m moves through a region of space where
figure: there are transverse mutually perpendicular electric and
B1  2R   0 j R 2 magnetic fields with strength E and induction B. If
the electric and magnetic fields are so adjusted that
B2  2( R  l )   0 j r 2
5 their effects cancel, and the beam passes through the
B  B1  B2 region without deviation then the electric force on the
B2
particles must be equal in magnitude and opposite in
B1 direction to the magnetic force. The electric force is
given by qE , and the magnetic force by qvB , where v
(viii) At point 6 in the is the velocity of the particles. Then,
figure: 6 qE  qvB,
B1
B1  2l   0 j  l 2 E
2
or v ,
B2  2(l 2)   0 j r B2 B
  O'
B  B1  B2

Vector B2 is
perpendicular to the line
O 6.

4
S
N2

P1 N2
+ A
x
N1 M N1
M

P2
a b
and this experiment can be used to measure the velocity a b
of the particles.
Next, the magnetic field is removed and the deflection Therefore, specific charge
of the particle caused by the electrostatic field is 2
E
measured as shown in the figure. The distance N1 N 2 is 2 x  
q N N v  B
the deflection caused by the electrostatic field as  1 2 
m MN1 Ea  a 
observed on a screen; P1 and P2 are the parallel plates,   b  Ea
 2 
and a is the length of the plates. As the particles
2 E x
traverse the field, they are subjected to a (vertical)  .
acceleration in a direction parallel to that of the field a (a  2b) B 2
a You should not have any difficulty in appreciating the
during the time interval , so that the velocity approximations applied in arriving at the above
v
produced in the direction of the electric field is equation. Strictly speaking, the horizontal distance the
beam covers outside the electric field is from the right
 qE  a  edge of the upper plate to the screen, which is equal to
 m  v  . On leaving the field, the velocity of the
   b. Instead, we have taken this horizontal distance as
particles has the components v in the horizontal a
 qE  a  qEa MN1   b . Why is it so? In the experimental setup,
2
direction and     in the vertical
 m  v  mv the distance of the screen from the plates is very large
direction. Hence, if N 2 is the deflected position of the as compared to the length a of the plates. Therefore,
N1 N 2 a
beam, is the ratio of the vertical and horizontal  b  b . But, even then, we could have taken the
MN1 2
velocities (how?): a
horizontal distance as b, instead of taking it as  b .
2
 qEa 
What made Thomson do so?
N1 N 2  mv 
 5. A ray of light traveling in air is incident at grazing
MN1 v
angle (incident angle = 90) on a long rectangular slab
q N1 N 2 v 2 of a transparent medium of thickness t = 1.0 m (see the
or  .
m MN1 Ea figure). The point of incidence is the origin A(0, 0). The
medium has a variable index of refraction n(y) given by
In equation, the quantities a , MN1 , and E are known
n(y) = [ky3/2 + 1]1/2,
from the experimental setup; the deflection N1 N 2 is where k = 1.0 (metre)-3/2.
measured, and v is known from the experiment in The refractive index of air is 1.0.
which the effects of the electric and magnetic fields
(a) Obtain a relation between the slope of the trajectory
cancelled each other.
of the ray at a point B(x, y) in the medium and the
Now we come to the problem. In the diagram given
incident angle at that point.
below (b) Obtain an equation for the trajectory y(x) of the ray
a in the medium.
N1 N 2  x, MN1   b .
2 (c) Determine the coordinates (x1, y1) of the point P,
where the ray intersects the upper surface of the slab-air
boundary.

5
y 1
Air P(x1, y1) 2
sin   . …(3)  dy 
2 1  
 dy   dx 
1  
 dx  1

t = 1.0 m
B(x, y)  dy 
 
 dx 
Medium
x Now from equation (2) and (3),
A(0, 0) Air
1 1 1
Solution: When a ray of light falls on a thin glass sheet,  
 dy 
2 n (ky  1)1/ 2
3/2
the emergent ray is parallel to the incident ray. This 1  
proposition can be easily demonstrated. Let us use this  dx 
fact in solving this problem. 2
 dy 
D or 1     (ky3/ 2  1)1/2
C Air
 dx 
2
Glass  dy 
B Air or 1+   = ky3/2 + 1
 dx 
AB || CD 2
 dy  3/2
A or   = ky
dx
 
Consider the glass slab as consisting of a large number dy
of sheets of infinitesimal thickness as shown in the or = k1/2 y3/4
figure below. These sheets have varying refractive dx
indices. Let us truncate the slab at point B. or y3/4dy = k1/2dx …(4)
If the given slab is truncated at B, the emergent ray from Integrating both sides of equation (4) with limits x = 0
point B will be parallel to the incident ray at point A. to x = x and y = 0 to y = y,
y x
Why? 3/ 4
dy   k1/2 dx
Let the ray after traveling through the medium fall on 0 y 0

surface CBD at . Let EB be the tangent to the path of y1/4


the ray at B. If the tangent makes an angle  with the or  k1/2 x
1/ 4
x-axis, slope at B equals tan 
k 2 x4
dy   or y . (Numerically k = 1).
(a) S  tan   tan      cot  …(1) 256
dx 2 
Second Solution: Consider a strip of infinitesimal
or S  cot  . thickness dy at y = y. Let the refractive index of the
y medium at point y = y be n. Let the refractive index
change by dn over dy.
y
P Air P(x1, y1)
C B 90
(x, y) D
t = 1.0 m.
 + d dy

Medium n + dn

x n
Air B(x, y)
A(0, 0) E 
y
Medium
(b) Applying Snell’s law at interface CBD, (with the x
slab truncated), A(0, 0) Air
n sin = 1  sin 90
Applying Snell’s law at the upper face of the strip.
1
or sin   . …(2) n sin = (n + dn) sin( + d)
n or n sin = (n + dn) (sin cos d + cos sin d)
dy or n sin = (n + dn) (sin + d cos)
From equation (1), = cot , which gives
dx or n sin = n sin + n d cos + dn sin + dn d cos

6
 cos d   1 
or dn sin = n cos d sin d   d  
 
 dn d  cos   0 
dn cos 
or  d .
n sin 
Integrating both sides of this equation from A to B,
n = 1 to n = n and  = /2 to  = ,
n dn  cos 
 1 n   / 2 sin  d 
n 
or ln n 1
  ln sin  / 2
or ln n =  ln sin
 1 
or ln n = ln  
 sin  
1
or n= .
sin 
Now proceed as in the previous solution.
Notice how smoothly and quickly we found n in the
previous method.
k 2 x4
(c) Equation of the trajectory is y = .
256
At point P, y = 1, x = 4.
Therefore, (x1, y1)  (4, 1).

Das könnte Ihnen auch gefallen